-
Notifications
You must be signed in to change notification settings - Fork 18
/
Copy pathchapter7.tex
466 lines (433 loc) · 27.9 KB
/
chapter7.tex
1
2
3
4
5
6
7
8
9
10
11
12
13
14
15
16
17
18
19
20
21
22
23
24
25
26
27
28
29
30
31
32
33
34
35
36
37
38
39
40
41
42
43
44
45
46
47
48
49
50
51
52
53
54
55
56
57
58
59
60
61
62
63
64
65
66
67
68
69
70
71
72
73
74
75
76
77
78
79
80
81
82
83
84
85
86
87
88
89
90
91
92
93
94
95
96
97
98
99
100
101
102
103
104
105
106
107
108
109
110
111
112
113
114
115
116
117
118
119
120
121
122
123
124
125
126
127
128
129
130
131
132
133
134
135
136
137
138
139
140
141
142
143
144
145
146
147
148
149
150
151
152
153
154
155
156
157
158
159
160
161
162
163
164
165
166
167
168
169
170
171
172
173
174
175
176
177
178
179
180
181
182
183
184
185
186
187
188
189
190
191
192
193
194
195
196
197
198
199
200
201
202
203
204
205
206
207
208
209
210
211
212
213
214
215
216
217
218
219
220
221
222
223
224
225
226
227
228
229
230
231
232
233
234
235
236
237
238
239
240
241
242
243
244
245
246
247
248
249
250
251
252
253
254
255
256
257
258
259
260
261
262
263
264
265
266
267
268
269
270
271
272
273
274
275
276
277
278
279
280
281
282
283
284
285
286
287
288
289
290
291
292
293
294
295
296
297
298
299
300
301
302
303
304
305
306
307
308
309
310
311
312
313
314
315
316
317
318
319
320
321
322
323
324
325
326
327
328
329
330
331
332
333
334
335
336
337
338
339
340
341
342
343
344
345
346
347
348
349
350
351
352
353
354
355
356
357
358
359
360
361
362
363
364
365
366
367
368
369
370
371
372
373
374
375
376
377
378
379
380
381
382
383
384
385
386
387
388
389
390
391
392
393
394
395
396
397
398
399
400
401
402
403
404
405
406
407
408
409
410
411
412
413
414
415
416
417
418
419
420
421
422
423
424
425
426
427
428
429
430
431
432
433
434
435
436
437
438
439
440
441
442
443
444
445
446
447
448
449
450
451
452
453
454
455
456
457
458
459
460
461
462
463
464
465
466
\section{Chapter VII.\hspace{0.2em} Fields}
\subsection{\textsection1. Field extensions, I}
\begin{problem}[1.1]
$\triangleright$ Prove that if $k \subseteq K$ is a field extension, then $\mathrm{char}\,k=\mathrm{char}\,K$. Prove that the category $\mathsf{Fld}$ has no initial object. $[\S 1.1]$
\end{problem}
\begin{solution}
Since $\mathbb{Z}$ is initial in $\mathsf{Ring}$, we have unique ring homomorphisms $i_k: \mathbb{Z} \rightarrow k$ and $i_K: \mathbb{Z} \rightarrow K$. Note that the inclusion $j:k\rightarrow K$ is a ring homomorphism, we have $i_K=j\circ i_k$. Since
\[
\ker i_K=\ker(j\circ i_k)=i_{k}^{-1}(\ker j)=i_{k}^{-1}(\{0\})=\ker i_k,
\]
we have $\mathrm{char}\,k=\mathrm{char}\,K$. If there exists one initial object $A$ in $\mathsf{Fld}$, all the objects in $\mathsf{Fld}$ will have the same characteristic as $A$. This contradicts with the fact that $\mathrm{char}\,{\mathbb{Z}}_2 \ne \mathrm{char}\,\mathbb{Z}_3$.
\end{solution}
\begin{problem}[1.3]
$\triangleright$ Let $k \subseteq F$ be a field extension, and let $\alpha \in F .$ Prove that the field $k(\alpha)$ consists of all the elements of $F$ which may be written as rational functions in $\alpha$, with coefficients in $k$. Why does this not give (in general) an onto homomorphism $k(t) \rightarrow k(\alpha) ?[\S 1.2, \S 1.3]$
\end{problem}
\begin{solution}
Since $k(\alpha)$ is smallest subfield of $F$ containing both $k$ and $\alpha$, for any $g(t)\in k(t)$ we have $g(\alpha)\in k(\alpha)$. Thus we can define the evaluation mapping
\begin{align*}
\mathrm{ev}_{\alpha}:k(t) &\longrightarrow k(\alpha)\\
g(t)&\longmapsto g(\alpha).
\end{align*}
It is clear to see $\mathrm{ev}_{\alpha}(k(t))\subseteq k(\alpha)$ and $k(\alpha)\subseteq \mathrm{ev}_{\alpha}(k(t))$, which means $k(\alpha)= \mathrm{ev}_{\alpha}(k(t))$.
If $\mathrm{ev}_{\alpha}:k(t) \rightarrow k(\alpha)$ is an onto field homomorphism, then $\mathrm{ev}_{\alpha}$ must be a field isomorphism. If we consider the simple extension $\mathbb{Q}\subseteq\mathbb{Q}(\sqrt{2})$, we will find that
\[
\mathrm{ev}_{\sqrt{2}}(0)=\mathrm{ev}_{\sqrt{2}}(t^2-1)=0,
\]
which contradicts the fact that $\mathrm{ev}_{\sqrt{2}}$ is an isomorphism.
\end{solution}
\begin{problem}[1.4]
Let $k \subseteq k(\alpha)$ be a simple extension, with $\alpha$ transcendental over $k$. Let $E$ be a subfield of $k(\alpha)$ properly containing $k$. Prove that $k(\alpha)$ is a finite extension of $E$.
\end{problem}
\begin{solution}
The field extension $E\subseteq k(\alpha)$ is finitely generated because $E(\alpha)= k(\alpha)$. Since $E$ be a subfield of $k(\alpha)$ properly containing $k$, we can suppose that there exist $f,g\in k[t]$ such that $g\ne0$, $\deg f>0$ and
\[
\frac{f(\alpha)}{g(\alpha)}\in E.
\]
Then let
\[
h(t)=f(t)-\frac{f(\alpha)}{g(\alpha)}g(t)\in E[t].
\]
It is immediate that $h(\alpha)=0$, which means $\alpha$ is algebraic over $E$. Thus we show that $E\subseteq E(\alpha)$ is a finite extension, or equivalently $k(\alpha)$ is a finite extension of $E$.
\end{solution}
\begin{problem}[1.5]
$\triangleright$ (Cf. Example 1.4.)
\begin{itemize}
\item Prove that there is exactly one subfield of $\mathbb{R}$ isomorphic to $\mathbb{Q}[t] /\left(t^{2}-2\right)$.
\item Prove that there are exactly three subfields of $\mathbb{C}$ isomorphic to $\mathbb{Q}[t] /\left(t^{3}-2\right)$.
\end{itemize}
From a `topological' point of view, one of these copies of $\mathbb{Q}[t] /\left(t^{3}-2\right)$ looks very different from the other two: it is not dense in $\mathbb{C}$, but the others are. $[\S 1.2]$
\end{problem}
\begin{solution}
\begin{itemize}
\item We will show that $\mathbb{Q}(\sqrt{2})$ is the unique subfield of $\mathbb{R}$ isomorphic to $\mathbb{Q}[t] /\left(t^{2}-2\right)$.
First we assert that $\mathbb{Q}[t] /\left(t^{2}-2\right)\cong\mathbb{Q}(\sqrt{2})$. To prove this, notice that $\mathbb{Q}[t]$ is a PID and $t^2-2$ is irreducible over $\mathbb{Q}[t]$, which implies $\left(t^{2}-2\right)$ is a maximal ideal of $\mathbb{Q}[t]$ and $\mathbb{Q}[t] /\left(t^{2}-2\right)$ is a field. Then we can check that
\begin{align*}
\varphi:\mathbb{Q}[t] /\left(t^{2}-2\right)&\longrightarrow \mathbb{Q}(\sqrt{2}) \\
p(t)+(t^2-2)&\longmapsto p(\sqrt{2})
\end{align*}
is an isomorphism.
Suppose that $F$ is a subfield of $\mathbb{R}$ and $\psi:\mathbb{Q}[t] /\left(t^{2}-2\right)\to F$ is an isomorphism. Let $\bar{t}=t+(t^2-2)\in \mathbb{Q}[t] /\left(t^{2}-2\right)$. Since $\psi(\bar{t})\in F\subseteq\mathbb{R}$, we have
\[
\psi\left(\bar{t}\right)^2-2=\psi\left(\overline{t}^2-2\right)=0\implies \psi(\bar{t})=\sqrt{2}\text{ or }-\sqrt{2}\implies \mathbb{Q}(\sqrt{2})\subseteq F.
\]
For any $x\in F$, there exists $g=q_1t+q_2+\left(t^{2}-2\right)\in\mathbb{Q}[t] /\left(t^{2}-2\right)$ such that $\psi(g)=x$, where $q_1,q_2\in\mathbb{Q}$. If $q_1=0$, we have
\[
x-q_2=\psi(g-q_2)=0\implies x=q_2\in \mathbb{Q}(\sqrt{2}).
\]
If $q_1\ne0$, we have
\[
(x-q_2)^2-2q_1^2=\psi((g-q_2)^2-2q_1^2)=\psi(q_1^2t^2-2q_1^2+(t^2-2))=0,
\]
which implies $x=\pm q_1\sqrt{2}+q_2\in \mathbb{Q}(\sqrt{2})$. Thus we have $F\subseteq\mathbb{Q}(\sqrt{2})$. Therefore $F=\mathbb{Q}(\sqrt{2})$, which guarantees the uniqueness of $\mathbb{Q}(\sqrt{2})$.
\item We can show that
\begin{align*}
\mathbb{Q}(\sqrt[3]{2}),\mathbb{Q}\left(\frac{-1+\sqrt{3}i}{2}\sqrt[3]{2}\right),\mathbb{Q}\left(\frac{-1-\sqrt{3}i}{2}\sqrt[3]{2}\right)
\end{align*}
are all isomorphic to $\mathbb{Q}[t] /\left(t^{3}-2\right)$. $\mathbb{Q}(\sqrt[3]{2})$ is not dense in $\mathbb{C}$.
\end{itemize}
\end{solution}
\begin{problem}[1.6]
$\triangleright$ Let $k \subseteq F$ be a field extension, and let $f(x) \in k[x]$ be a polynomial. Prove that $\operatorname{Aut}_{k}(F)$ acts on the set of roots of $f(x)$ contained in $F$. Provide examples showing that this action need not be transitive or faithful. [\S1.2, \S1.3]
\end{problem}
\begin{solution}
Let $S_f$ be the set of roots of $f(x)$ contained in $F$. Note that for any $\sigma\in\operatorname{Aut}_{k}(F)$ and any $\alpha\in S_f$, $\sigma$ can be extend to $F[x]$ and $\sigma(f)=f$. Since
\[
f(\sigma(\alpha))=\sigma(f)(\sigma(\alpha))=\sigma(f(\alpha))=\sigma(0)=0\implies\sigma(\alpha)\in S_f,
\]
we can define the mapping
\begin{align*}
\cdot:\operatorname{Aut}_{k}(F)\times S_f &\longrightarrow S_f\\
(\sigma, \alpha) &\longmapsto \sigma\cdot\alpha:=\sigma(\alpha).
\end{align*}
Since for any $\sigma_1,\sigma_2\in\operatorname{Aut}_{k}(F)$ and any root $\alpha$ of $f(x)$,
\[
\sigma_1\cdot(\sigma_2\cdot\alpha)=\sigma_1\cdot\sigma_2(\alpha)=\sigma_1(\sigma_2(\alpha))=(\sigma_1\circ\sigma_2)(\alpha)=(\sigma_1\circ\sigma_2)\cdot\alpha,
\]
we see that $\operatorname{Aut}_{k}(F)$ acts on the set of roots of $f(x)$ contained in $F$.
\end{solution}
\hypertarget{Exercise VII.1.7}{}
\begin{problem}[1.7]
Let $k \subseteq F$ be a field extension, and let $\alpha \in F$ be algebraic over $k$.
\begin{itemize}
\item Suppose $p(x) \in k[x]$ is an irreducible monic polynomial such that $p(\alpha)=0$; prove that $p(x)$ is the minimal polynomial of $\alpha$ over $k$, in the sense of Proposition 1.3.
\item Let $f(x) \in k[x]$. Prove that $f(\alpha)=0$ if and only if $p(x) \mid f(x)$.
\item Show that the minimal polynomial of $\alpha$ is the minimal polynomial of a certain $k$-linear transformation of $F$, in the sense of Definition $\mathrm{VI}.6.12$.
\end{itemize}
\end{problem}
\begin{solution}
\begin{itemize}
\item Suppose $q(x)$ is the minimal polynomial of $\alpha$ over $k$. Since $\deg q(x)\le \deg p(x)$, we have Euclidean division in the Euclidean domain $k[x]$ as follows
\[
p(x) = m(x)q(x) + r(x),\quad \deg r(x)<\deg q(x).
\]
Taking $x=\alpha$, there must be $r(\alpha)=0$. Since $q(x)$ is the minimal polynomial, we can assert $r(x)=0$ and accordingly $q(x)\mid p(x)$. The irreducibility of $p(x)$ means $p(x) = q(x)$.
\item It is clear that $p(x) \mid f(x)\implies f(\alpha)=p(\alpha)h(\alpha)=0$. To show $f(\alpha)=0\implies p(x) \mid f(x)$, we can just follow the same procedure as the first problem.
\item Suppose $p(x) \in k[x]$ is the minimal polynomial of $\alpha$. We can easily check that
\begin{align*}
T_\alpha:F&\longrightarrow F,\\
m&\longmapsto\alpha m
\end{align*}
is a $k$-linear transformation. Since for any $m\in F$,
\begin{align*}
p(T_\alpha)(m)&=c_0+c_1T_\alpha m+c_2T_\alpha^2 m+\cdots+c_nT_\alpha^n m\\
&=c_0+c_1\alpha m+c_2\alpha^2 m+\cdots+c_n\alpha^n m\\
&=\left(c_0+c_1\alpha +c_2\alpha^2 +\cdots+c_n\alpha^n\right) m=0,
\end{align*}
we have $p(T_\alpha)=0$. Since $p(x)$ is an irreducible monic polynomial, $p(x)$ is exactly the minimal polynomial of $T_\alpha$.
\end{itemize}
\end{solution}
\begin{problem}[1.8]
$\neg$ Let $f(x) \in k[x]$ be a polynomial over a field $k$ of degree $d$, and let $\alpha_{1}, \ldots, \alpha_{d}$ be the roots of $f(x)$ in an extension of $k$ where the polynomial factors completely. For a subset $I \subseteq\{1, \ldots, d\}$, denote by $\alpha_{I}$ the sum $\sum_{i \in I} \alpha_{i}$. Assume that $\alpha_{I} \in k$ only for $I=\emptyset$ and $I=\{1, \ldots, d\}$. Prove that $f(x)$ is irreducible over $k$. [7.14]
\end{problem}
\begin{solution}
Suppose $f(x)=(x-\alpha_1)\cdots(x-\alpha_d)$ is reducible over $k$ and $g(x)=\prod_{i\in I} (x-\alpha_{i})\in k[x]$ is a factor of $f$, where $I\subsetneq \{1, \ldots, d\}$. Note the coefficient of the $n-1$-th degree term of $g(x)$ is
\[
-\sum_{i \in I} \alpha_{i}\notin k.
\]
We derive a contradiction. Hence we can conclude that $f(x)$ is irreducible over $k$.
\end{solution}
\begin{problem}[1.10]
$\neg$ Let $k$ be a field. Prove that the ring of square $n \times n$ matrices $\mathcal{M}_{n}(k)$ contains an isomorphic copy of every extension of $k$ of degree $\leq n$. (Hint: If $k \subseteq F$ is an extension of degree $n$ and $\alpha \in F$, then `multiplication by $\alpha$' is a $k$-linear transformation of $F$.) [5.20]
\end{problem}
\begin{solution}
Suppose $k\subseteq F$ is an extension of degree $n$ and $\alpha \in F$, then
\begin{align*}
T_\alpha:F&\longrightarrow F,\\
m&\longmapsto\alpha m
\end{align*}
is a $k$-linear transformation of $F$, which is isomorphic to a matrix $A_{T_\alpha}\in \mathcal{M}_{n}(k)$ in $\mathsf{Vect}_k$. What is left to do is to show that $A_{T_{\alpha+\beta}}=A_{T_{\alpha}}+A_{T_{\beta}}$, $A_{T_{\alpha\beta}}=A_{T_{\alpha}}A_{T_{\beta}}$ and $A_{T_{1_F}}=I_{n\times n}$, which amounts to $T_{\alpha\beta}=T_{\alpha}+T_{\beta}$, $T_{\alpha\beta}=T_{\alpha}T_{\beta}$ and $T_{1_F}=\mathrm{id}_F$. The verification is straightforward.
\end{solution}
\hypertarget{Exercise VII.1.11}{}
\begin{problem}[1.11]
$\neg$ Let $k \subseteq F$ be a finite field extension, and let $p(x)$ be the characteristic polynomial of the $k$-linear transformation of $F$ given by multiplication by $\alpha$. Prove that $p(\alpha)=0$.
\hspace{2em}This gives an effective way to find a polynomial satisfied by an element of an extension. Use it to find a polynomial satisfied by $\sqrt{2}+\sqrt{3}$ over $\mathbb{Q}$, and compare this method with the one used in Example 1.19. [1.12]
\end{problem}
\begin{solution}
In \hyperlink{Exercise VII.1.7}{Exercise VII.1.7} we show that the minimal polynomial of $\alpha$ coincides with the minimal polynomial of $T_\alpha$. Denote the minimal polynomial as $f(x)$. Then we have $f(\alpha)=0$ and $f(x)\mid p(x)$, which implies $p(\alpha)=0$.
Consider the field extension $\mathbb{Q}\subseteq\mathbb{Q}(\sqrt{2},\sqrt{3})$ and the algebraic element $\alpha=\sqrt{2}+\sqrt{3}$. Since $1, \sqrt{2}, \sqrt{3}, \sqrt{6}$ is a basis of $\mathbb{Q}(\sqrt{2},\sqrt{3})$, we can represent the linear transformation by a matrix $A$ as follows
\begin{align*}
&\hspace{15pt}\left(T_\alpha 1,T_\alpha\sqrt{2},T_\alpha\sqrt{3},T_\alpha\sqrt{6}\right)\\
&=\left(\sqrt{2}+\sqrt{3},2+\sqrt{6},3+\sqrt{6},3\sqrt{2}+2\sqrt{3}\right)\\
&=\left(1,\sqrt{2},\sqrt{3},\sqrt{6}\right)
\begin{pmatrix}
0&2&3&0\\
1&0&0&3\\
1&0&0&2\\
0&1&1&0
\end{pmatrix}\\
&=\left(1,\sqrt{2},\sqrt{3},\sqrt{6}\right)A.
\end{align*}
The characteristic polynomial of $A$ is
\[
p(x)=\det(xI-A)=x^4-10x^2+1.
\]
\end{solution}
\hypertarget{Exercise VII.1.12}{}
\begin{problem}[1.12]
$\neg$ Let $k \subseteq F$ be a finite field extension, and let $\alpha \in F$. The norm of $\alpha$, $N_{k \subseteq F}(\alpha)$, is the determinant of the linear transformation of $F$ given by multiplication by $\alpha$ (cf. \hyperlink{Exercise VII.1.11}{Exercise VII.1.11}, Definition VI.6.4).
Prove that the norm is multiplicative: for $\alpha, \beta \in F$,
$$
N_{k \subseteq F}(\alpha \beta)=N_{k \subseteq F}(\alpha) N_{k \subseteq F}(\beta) .
$$
Compute the norm of a complex number viewed as an element of the extension $\mathbb{R} \subseteq \mathbb{C}$ (and marvel at the excellent choice of terminology). Do the same for elements of an extension $\mathbb{Q}(\sqrt{d})$ of $\mathbb{Q}$, where $d$ is an integer that is not a square, and compare the result with \hyperlink{Exercise III.4.10}{Exercise III.4.10}. [1.13, 1.14, 1.15, 4.19, 6.18, VIII.1.5]
\end{problem}
\begin{solution}
consider the field extension $\mathbb{R} \subseteq \mathbb{C}$ and a complex number $w=a+bi\in \mathbb{C}$. Given a basis $1$, $i$ of $\mathbb{C}$, denote the matrix representation of the linear transformation $F:z\mapsto wz$ as $A$. Then we have
\begin{align*}
&\hspace{15pt}\left(F(1),F(i)\right)\\
&=\left(a+bi,-b+ai\right)\\
&=\left(1,i\right)
\begin{pmatrix}
a&-b\\
b&a\\
\end{pmatrix}\\
&=\left(1,i\right)A.
\end{align*}
The norm $N_{\mathbb{R} \subseteq \mathbb{C}}(w)=\det(A)=a^2+b^2$.
consider the field extension $\mathbb{Q} \subseteq \mathbb{Q}\left(\sqrt{d}\right)$ and a number $c=a+b\sqrt{d}\in \mathbb{Q}\left(\sqrt{d}\right)$. Given a basis $1$, $\sqrt{d}$ of $ \mathbb{Q}\left(\sqrt{d}\right)$, denote the matrix representation of the linear transformation $F:x\mapsto cx$ as $B$. Then we have
\begin{align*}
&\hspace{15pt}\left(F(1),F\left(\sqrt{d}\right)\right)\\
&=\left(a+b\sqrt{d},bd+a\sqrt{d}\right)\\
&=\left(1,\sqrt{d}\right)
\begin{pmatrix}
a&bd\\
b&a\\
\end{pmatrix}\\
&=\left(1,\sqrt{d}\right)B.
\end{align*}
The norm $N_{\mathbb{Q} \subseteq \mathbb{Q}(\sqrt{d})}(c)=\det(B)=a^2-db^2$.
\end{solution}
\hypertarget{Exercise VII.1.13}{}
\begin{problem}[1.13]
$\neg$ Define the trace $\operatorname{tr}_{k \subseteq F}(\alpha)$ of an element $\alpha$ of a finite extension $F$ of a field $k$ by following the lead of \hyperlink{Exercise VII.1.12}{Exercise VII.1.12}. Prove that the trace is additive:
$$
\operatorname{tr}_{k \subseteq F}(\alpha+\beta)=\operatorname{tr}_{k \subseteq F}(\alpha)+\operatorname{tr}_{k \subseteq F}(\beta)
$$
for $\alpha, \beta \in F$. Compute the trace of an element of an extension $\mathbb{Q} \subseteq \mathbb{Q}(\sqrt{d})$, for $d$ an integer that is not a square. [1.14, 1.15, 4.19, VIII.1.5]
\end{problem}
\begin{solution}
Given $c=a+b\sqrt{d}\in \mathbb{Q}\left(\sqrt{d}\right)$, the trace $\mathrm{tr}_{\mathbb{Q} \subseteq \mathbb{Q}(\sqrt{d})}(c)=\operatorname{tr}(B)=2a$.
\end{solution}
\begin{problem}[1.14]
$\neg$ Let $k \subseteq k(\alpha)$ be a simple algebraic extension, and let $x^{d}+a_{d-1} x^{d-1}+\cdots+a_{0}$ be the minimal polynomial of $\alpha$ over $k$. Prove that
$$
\operatorname{tr}_{k \subseteq k(\alpha)}(\alpha)=-a_{d-1} \quad \text { and } \quad N_{k \subseteq k(\alpha)}(\alpha)=(-1)^{d} a_{0} .
$$
(Cf. \hyperlink{Exercise VII.1.12}{Exercise VII.1.12} and \hyperlink{Exercise VII.1.13}{Exercise VII.1.13}). [4.19]
\end{problem}
\begin{solution}
Since $p(x)=x^{d}+a_{d-1} x^{d-1}+\cdots+a_{0}$ is the minimal polynomial of $\alpha$, we see $1,\alpha,\cdots,\alpha^{d-1}$ is the basis of the $k$-vector space $k(\alpha)$ and
\begin{align*}
p(\alpha)=\alpha^{d}+a_{d-1} \alpha^{d-1}+\cdots+a_{0}=0
\end{align*}
The matrix representation of the linear map $F:z\mapsto \alpha z$ is
\begin{align*}
&\hspace{15pt}\left(F(1),F\left(\alpha\right),\cdots,F\left(\alpha^{d-1}\right)\right)\\
&=\left(1,\alpha,\alpha^2,\cdots,\alpha^{d-1}\right)
\begin{pmatrix}
0&0&0&\cdots&0&-a_0\\
1&0&0&\cdots&0&-a_1\\
0&1&0&\cdots&0&-a_2\\
\vdots&\vdots&\vdots&&\vdots&\vdots\\
0&0&0&\cdots&0&-a_{d-2}\\
0&0&0&\cdots&1&-a_{d-1}
\end{pmatrix}\\
&=\left(1,\alpha\right)A.
\end{align*}
Thus we have
\begin{align*}
\operatorname{tr}_{k \subseteq k(\alpha)}(\alpha)=\operatorname{tr}(A)=-a_{d-1}
\end{align*}
and
\begin{align*}
N_{k \subseteq k(\alpha)}(\alpha)=\det(A)=(-1)^{d-1} (-a_{0})= (-1)^{d} a_{0}.
\end{align*}
\end{solution}
\begin{problem}[1.16]
$\triangleright$ Let $k \subseteq L \subseteq F$ be fields, and let $\alpha \in F$. If $k \subseteq k(\alpha)$ is a finite extension, then $L \subseteq L(\alpha)$ is finite and $[L(\alpha): L] \leq[k(\alpha): k]$. [§1.3]
\end{problem}
\begin{solution}
Since $k \subseteq k(\alpha)$ is a finite extension, it is a simple algebraic extension and there exists a minimal polynomial $p(t)$ of $\alpha$ over $k$. Since $p(t)$ can be seen as a polynomial in $L$ and $p(\alpha)=0$, we have $L \subseteq L(\alpha)$ is finite. Since the degree of the minimal polynomial of $\alpha$ over $L$ is not greater than the degree of $p(t)$, we have $[L(\alpha): L] \leq[k(\alpha): k]$.
\end{solution}
\begin{problem}[1.20]
Let $p$ be a prime integer, and let $\alpha=\sqrt[p]{2} \in \mathbb{R}$. Let $g(x) \in \mathbb{Q}[x]$ be any nonconstant polynomial of degree $<p$. Prove that $\alpha$ may be expressed as a polynomial in $g(\alpha)$ with rational coefficients.\\
Prove that an analogous statement for $\sqrt[4]{2}$ is false.
\end{problem}
\begin{solution}
It is easy to see that $\mathbb{Q}\subseteq \mathbb{Q}(g(\alpha)) \subseteq\mathbb{Q}(\alpha)$. Since $f(t)=x^p-2$ is irredicible over $\mathbb{Q}$ and $f(\alpha)=0$, $\mathbb{Q}\subseteq \mathbb{Q}(\alpha)$ is a $p$ degree extension. If $p$ is a prime integer, then the degree fo the finite extension $\mathbb{Q}\subseteq \mathbb{Q}(g(\alpha))$ is 1 or $p$. Note that $g(\alpha)\notin \mathbb{Q}$. There must be $\mathbb{Q}(g(\alpha))=\mathbb{Q}(\alpha)$. Thus we see there exists $h\in \mathbb{Q}[x]$ such that $\alpha = h(g(\alpha))$ with rational coefficients.
If $p=4$, then we can take $g(x)=x^2$. Now we have $g(\alpha)=\sqrt{2}$. For the tower of field extensions $\mathbb{Q}\subseteq \mathbb{Q}(\sqrt{2}) \subseteq\mathbb{Q}(\sqrt[4]{2})$, it is easy to check that both $\mathbb{Q}\subseteq \mathbb{Q}(\sqrt{2}) $ and $ \mathbb{Q}(\sqrt{2}) \subseteq\mathbb{Q}(\sqrt[4]{2})$ are quadratic extensions. Hence the intermediate field $ \mathbb{Q}(\sqrt{2})$ is not equal to $\mathbb{Q}$ or $\mathbb{Q}(\sqrt[4]{2})$. Therefore, we can conclude that $\alpha=\sqrt[4]{2}$ cannot be expressed as a polynomial in $g(\alpha)=\sqrt{2}$ with rational coefficients.
\end{solution}
\begin{problem}[1.22]
Let $k \subseteq F$ be a field extension, and let $\alpha \in F, \beta \in F$ be algebraic, of degree $d, e$, resp. Assume $d, e$ are relatively prime, and let $p(x)$ be the minimal polynomial of $\beta$ over $k$. Prove $p(x)$ is irreducible over $k(\alpha)$.
\end{problem}
\begin{solution}
By the tower properties of finite field extensions $k \subseteq k(\alpha) \subseteq k(\alpha,\beta)$ and $k \subseteq k(\beta) \subseteq k(\alpha,\beta)$, we have $d$ divides $[k(\alpha,\beta): k]$ and $e$ divides $[k(\alpha,\beta): k]$. Since $d$ and $e$ are relatively prime, we have $[k(\alpha,\beta): k]\ge de$. Since $p(\beta)=0$, the minimal polynomial of $\beta$ over $k(\alpha)$ divides $p(x)$, which implies $ [k(\alpha)(\beta): k(\alpha)]\le \deg(p)=e$. By the tower property of finite field extensions $k \subseteq k(\alpha) \subseteq k(\alpha,\beta)$, we have $[k(\alpha,\beta): k]\le de$, which forces $[k(\alpha,\beta): k]= de$ and $[k(\alpha)(\beta): k(\alpha)]=\deg(p)=e$. Thus we see $p(x)$ is the minimal polynomial of $\beta$ over $k(\alpha)$, and hence $p(x)$ is irreducible over $k(\alpha)$.
\end{solution}
\begin{problem}[1.23]
Express $\sqrt{2}$ explicitly as a polynomial function in $\sqrt{2}+\sqrt{3}$ with rational coefficients.
\end{problem}
\begin{solution}
Suppose $f(x)=a+b x+c x^2+d x^3$ is a polynomial in $\mathbb{Q}[x]$.
\begin{align*}
&\;a+b\left(\sqrt{2}+\sqrt{3}\right)+c\left(\sqrt{2}+\sqrt{3}\right)^2+d\left(\sqrt{2}+\sqrt{3}\right)^3\\
=&\;a+b\left(\sqrt{2}+\sqrt{3}\right)+c\left(5+2\sqrt{6}\right)+d\left(11\sqrt{2}+9\sqrt{3}\right)\\
=&\;a+5c+(b+11d)\sqrt{2}+(b+9d)\sqrt{3}+2c \sqrt{6}
\end{align*}
Solve
\begin{align*}
a+5c&=0\\
b+11d&=1\\
b+9d&=0\\
2c&=0
\end{align*}
and we get $a=0, b=-\frac{9}{2}, c=0, d=\frac{1}{2}$. Therefore, $\sqrt{2}$ can be expressed as
\[
\sqrt{2}=f\left(\sqrt{2}+\sqrt{3}\right)=\frac{1}{2}\left(\sqrt{2}+\sqrt{3}\right)^3-\frac{9}{2}\left(\sqrt{2}+\sqrt{3}\right) .
\]
\end{solution}
\begin{problem}[1.25]
$\neg$ Let $\xi:=\sqrt{2+\sqrt{2}}$.
\begin{itemize}
\item Find the minimal polynomial of $\xi$ over $\mathbb{Q}$, and show that $\mathbb{Q}(\xi)$ has degree 4 over $\mathbb{Q}$.
\item Prove that $\sqrt{2-\sqrt{2}}$ is another root of the minimal polynomial of $\xi$.
\item Prove that $\sqrt{2-\sqrt{2}} \in \mathbb{Q}(\xi)$. (Hint: $\left.(a+b)(a-b)=a^2-b^2.\right)$
\item By Proposition 1.5, sending $\xi$ to $\sqrt{2-\sqrt{2}}$ defines an automorphism of $\mathbb{Q}(\xi)$ over $\mathbb{Q}$. Find the matrix of this automorphism w.r.t. the basis $1, \xi, \xi^2, \xi^3$.
\item Prove that $\operatorname{Aut}_{\mathbb{Q}}(\mathbb{Q}(\xi))$ is cyclic of order 4.
\end{itemize}
[6.6]
\end{problem}
\begin{solution}
\begin{itemize}
\item Let
\[
p(x)=\left(x^2-2\right)^2-2=x^4-4x^2+2.
\]
By Eisenstein's criterion, $p(x)$ is irreducible over $\mathbb{Q}$. Since $p(\xi)=0$, $p(x)$ is the minimal polynomial of $\xi$ over $\mathbb{Q}$. Therefore, $\mathbb{Q}(\xi)$ has degree 4 over $\mathbb{Q}$.
\item It is straightforward to check that $p\left(\sqrt{2-\sqrt{2}}\right)=0$.
\item
\begin{align*}
\sqrt{2-\sqrt{2}}=\sqrt{\frac{\left(2-\sqrt{2}\right)\left(2+\sqrt{2}\right)}{2+\sqrt{2}}}=\frac{\sqrt{2}}{\sqrt{2+\sqrt{2}}}=\frac{\xi^2-2}{\xi}
\end{align*}
\item Suppose $\sigma$ is an automorphism of $\mathbb{Q}(\xi)$ over $\mathbb{Q}$ sending $\xi$ to $\sqrt{2-\sqrt{2}}$. We need express $\sigma(1),\sigma(\xi),\sigma(\xi^2),\sigma(\xi^3)$ as the linear combination of $1, \xi, \xi^2, \xi^3$. Note
\begin{align*}
\sigma(\xi)&=\sqrt{2-\sqrt{2}}=\xi-\frac{2}{\xi}.
\end{align*}
We may express $\frac{1}{\xi}$ as a linear combination of $\xi$ and $\xi^3$. Suppose there exists $a,b\in \mathbb{Q}$ such that
\[
\frac{1}{\xi}=a\xi+b\xi^3.
\]
Then we have
\begin{align*}
&\;a\xi^2+b\xi^4\\
=&\;a(2+\sqrt{2})+b(6+4\sqrt{2})\\
=&\;2a+6b+(a+4b)\sqrt{2}\\
=&\;1,
\end{align*}
which implies $a=2$, $b=-\frac{1}{2}$ and $\frac{1}{\xi}=2\xi-\frac{1}{2}\xi^3$. Therefore,
\begin{align*}
\sigma(\xi)&=\xi-\frac{2}{\xi}=\xi-2\left(2\xi-\frac{1}{2}\xi^3\right)=-3\xi+\xi^3\\
\sigma\left(\xi^2\right)&=\sigma(\xi)^2=2-\sqrt{2}=4-\xi^2\\
\sigma\left(\xi^3\right)&=\left(\xi-\frac{2}{\xi}\right)\left(4-\xi^2\right)=-\xi^3+6 \xi-\frac{8}{\xi}=-\xi^3+6 \xi-8\left(2\xi-\frac{1}{2}\xi^3\right)=-10\xi+3\xi^3.
\end{align*}
and the matrix representation of $\sigma$ w.r.t. the basis $1, \xi, \xi^2, \xi^3$ is
\begin{align*}
&\hspace{15pt}\left(\sigma(1),\sigma(\xi),\sigma(\xi^2),\sigma(\xi^3)\right)\\
&=\left(1, \xi, \xi^2, \xi^3\right)
\begin{pmatrix}
1&0&4&0\\
0&-3&0&-10\\
0&0&-1&0\\
0&1&0&3
\end{pmatrix}\\
&=\left(1, \xi, \xi^2, \xi^3\right)A.
\end{align*}
\item Since
\[
x_1=\sqrt{2+\sqrt{2}},\quad x_2=\sqrt{2-\sqrt{2}},\quad x_3=-\sqrt{2+\sqrt{2}},\quad x_4=-\sqrt{2-\sqrt{2}}
\]
are the all roots of $p(x)$ over $\overline{\mathbb{Q}}$ and $\mathbb{Q}(\xi)=\mathbb{Q}(x_1,x_2,x_3,x_4)$, we see $\mathbb{Q}\subseteq\mathbb{Q}(\xi)$ is a Galois extension and the Galois group $\operatorname{Aut}_{\mathbb{Q}}(\mathbb{Q}(\xi))$ is of order 4. By calculating the characteristic polynomial of $A$
\begin{align*}
\det\left(\lambda I-A\right)&=\det \begin{pmatrix}
\lambda-1&0&-4&0\\
0&\lambda+3&0&10\\
0&0&\lambda+1&0\\
0&-1&0&\lambda-3
\end{pmatrix}\\
&=(\lambda-1)\left((\lambda+3)(\lambda+1)(\lambda-3)+10(\lambda+1)\right)\\
&=(\lambda-1)(\lambda+1)\left(\lambda^2+1\right)\\
&=\lambda^4-1,
\end{align*}
we have $A^4=I$. Since $A^2\ne I$, the order of $\sigma\in \operatorname{Aut}_{\mathbb{Q}}(\mathbb{Q}(\xi))$ is 4. Therefore, $\operatorname{Aut}_{\mathbb{Q}}(\mathbb{Q}(\xi))$ is a cyclic group of order 4.
\end{itemize}
\end{solution}
\subsection{\textsection3. Geometric impossibilities}
\begin{problem}[3.14]
Prove that the regular 9-gon is not constructible.|
\end{problem}
\begin{solution}
Suppose $\xi=e^{2\pi i/9}$. We can check that the 9th cyclotomic polynomial
\[
\Phi_9(x)=x^6+x^3+1,
\]
is the minimal polynomial $\xi$ over $\mathbb{Q}$. Therefore, $\mathbb{Q}(\xi)$ is a extension of degree 6 over $\mathbb{Q}$. Since 6 is not a power of 2, the regular 9-gon is not constructible.
\end{solution}
\begin{problem}[3.15]
$\triangleright$ Prove that if $2^k+1$ is prime, then $k$ is a power of 2. [\textsection3.3]
\end{problem}
\begin{solution}
If $k=0$, the proposition is trivial. If $k\ne0$, we can suppose $k=n2^m$ where $n$ is an odd number. Since
\[
2^k+1=\left(2^{2^m}\right)^n+1=\left(2^{2^m}+1\right)\left(2^{(n-1)2^m}-2^{(n-2)2^m}+\cdots+2^{2\cdot 2^m}-2^{ 2^m}+1\right)
\]
is prime, there msut be $n=1$, which implies $k$ is a power of 2.
\end{solution}
\subsection{\textsection4. Field extensions, II}
\begin{problem}[4.2]
Describe the splitting field of $x^6+x^3+1$ over $\mathbb{Q}$. Do the same for $x^4+4$.
\end{problem}
\begin{solution}
Suppose $\xi=e^{\frac{2\pi i}{9}}$. Then $x^6+x^3+1$ is the 9th cyclotomic polynomial
\[
\Phi_9(x)=x^6+x^3+1=\left(x-\xi\right)\left(x-\xi^2\right)\left(x-\xi^4\right)\left(x-\xi^5\right)\left(x-\xi^7\right)\left(x-\xi^8\right).
\]
The splitting field of $\Phi_9(x)$ over $\mathbb{Q}$ is $\mathbb{Q}(\xi,\xi^2,\xi^4,\xi^5,\xi^7,\xi^8)$.
\begin{align*}
x^4+4=(x^2+2i)(x^2-2i)=(x-(1+i))(x-(-1-i))(x-(-1+i))(x-(1-i)).
\end{align*}
The splitting field of $x^4+4$ over $\mathbb{Q}$ is $\mathbb{Q}(1+i,-1-i,-1+i,1-i)=\mathbb{Q}(i)$.
\end{solution}
\begin{problem}[4.3]
$\triangleright$ Find the order of the automorphism group of the splitting field of $x^4+2$ over $\mathbb{Q}$ (cf. Example 4.6). [\$4.1]
\end{problem}
\begin{solution}
\[
x^4+2=(x^2+\sqrt{2}i)(x^2-\sqrt{2}i)=\left(x-\frac{1+i}{\sqrt[4]{2}}\right)\left(x-\frac{-1-i}{\sqrt[4]{2}}\right)\left(x-\frac{-1+i}{\sqrt[4]{2}}\right)\left(x-\frac{1-i}{\sqrt[4]{2}}\right).
\]
The splitting field of $x^4+2$ over $\mathbb{Q}$ is $\mathbb{Q}\left(\frac{1+i}{\sqrt[4]{2}},\frac{-1-i}{\sqrt[4]{2}},\frac{-1+i}{\sqrt[4]{2}},\frac{1-i}{\sqrt[4]{2}}\right)=\mathbb{Q}\left(\sqrt[4]{2},i\right)$.
For the field extension $\mathbb{Q}\subseteq\mathbb{Q}(\sqrt[4]{2})$, since the minimal polynomial of $\sqrt[4]{2}$ over $\mathbb{Q}$ is $x^4-2$, we have $[\mathbb{Q}(\sqrt[4]{2}): \mathbb{Q}]=4$. For the field extension $\mathbb{Q}(\sqrt[4]{2})\subseteq\mathbb{Q}(\sqrt[4]{2},i)$, since the minimal polynomial of $i$ over $\mathbb{Q}(\sqrt[4]{2})$ is $x^2+1$, we have $[\mathbb{Q}(\sqrt[4]{2},i): \mathbb{Q}(\sqrt[4]{2})]=2$. By the tower property, we have
\[
[\mathbb{Q}(\sqrt[4]{2},i): \mathbb{Q}]=[\mathbb{Q}(\sqrt[4]{2},i): \mathbb{Q}(\sqrt[4]{2})][\mathbb{Q}(\sqrt[4]{2}): \mathbb{Q}]=2\times 4=8.
\]
Since $\mathbb{Q}\subseteq\mathbb{Q}(\sqrt[4]{2},i)$ is a Galois extension, the order of the Galois group $\operatorname{Aut}_{\mathbb{Q}}\left(\mathbb{Q}(\sqrt[4]{2},i)\right)$ is 8.
\end{solution}
\begin{problem}[4.4]
Prove that the field $\mathbb{Q}(\sqrt[4]{2})$ is not the splitting field of any polynomial over $\mathbb{Q}$.
\end{problem}
\begin{solution}
The polynomial $p(x)=x^4-2\in\mathbb{Q}[x]$ has root over $\mathbb{Q}(\sqrt[4]{2})$ because $p\left(\sqrt[4]{2}\right)=0$. However,
\[
p(x)=\left(x-\sqrt[4]{2}\right)\left(x+\sqrt[4]{2}\right)\left(x-\sqrt[4]{2}i\right)\left(x+\sqrt[4]{2}i\right)
\]
does not split over $\mathbb{Q}(\sqrt[4]{2})$ because $\sqrt[4]{2}i\notin \mathbb{Q}(\sqrt[4]{2})$, which implies that $\mathbb{Q}\subseteq\mathbb{Q}(\sqrt[4]{2})$ is not a normal extension and is not the splitting field of any polynomial over $\mathbb{Q}$.
\end{solution}